Difference between revisions of "2025 SSMO Speed Round Problems/Problem 7"
(Created page with "==Problem== Positive integers <math>a</math> and <math>b</math> satisfy <math>63a = 40b</math>. The sum of all possible values of <math>\tfrac{\varphi(a)}{\varphi(b)}</math>...") |
|||
Line 4: | Line 4: | ||
==Solution== | ==Solution== | ||
+ | |||
+ | Let <math>a = 2^{w+3}3^x5^{y+1}7^zT</math> and <math>b = 2^w3^{x+2}5^y7^{z+1}T</math>, where <math>w</math>, <math>x</math>, <math>y</math>, and <math>z</math> are all nonnegative integers and <math>T</math> is a positive integer not divisible by any of <math>2</math>, <math>3</math>, <math>5</math>, and <math>7</math>. Then, | ||
+ | <cmath>\frac{\varphi(a)}{\varphi(b)} = \frac{\varphi(2^{w+3})}{\varphi(2^{w})} \cdot \frac{\varphi(3^{x})}{\varphi(3^{x+2})} \cdot \frac{\varphi(5^{y+1})}{\varphi(5^{y})} \cdot \frac{\varphi(7^{z})}{\varphi(7^{z+1})}.</cmath>Now, we determine the possible values of each factor on the right hand side based on the values of <math>w</math>, <math>x</math>, <math>y</math>, and <math>z</math>. | ||
+ | <UL><LI>If <math>w=0</math>, then <math>\tfrac{\varphi(2^{w+3})}{\varphi(2^{w})} = 4</math>; if <math>w>0</math>, then <math>\tfrac{\varphi(2^{w+3})}{\varphi(2^{w})} = \tfrac{2^{w+2}\varphi(2)}{2^{w-1}\varphi(2)} = 8</math>.</LI> | ||
+ | <LI>If <math>x=0</math>, then <math>\tfrac{\varphi(3^{x})}{\varphi(3^{x+2})} = \tfrac{1}{6}</math>; if <math>x>0</math>, then <math>\tfrac{\varphi(3^{x})}{\varphi(3^{x+2})} = \tfrac{3^{x-1}\varphi(3)}{3^{x+1}\varphi(3)} = \tfrac{1}{9}</math>.</LI> | ||
+ | <LI>If <math>y=0</math>, then <math>\tfrac{\varphi(5^{y+1})}{\varphi(5^{y})} = 4</math>; if <math>y>0</math>, then <math>\tfrac{\varphi(5^{y+1})}{\varphi(5^{y})} = \tfrac{5^{y}\varphi(5)}{5^{y-1}\varphi(5)} = 5</math>.</LI> | ||
+ | <LI>If <math>z=0</math>, then <math>\tfrac{\varphi(7^{z})}{\varphi(7^{z+1})} = \tfrac{1}{6}</math>; if <math>z>0</math>, then <math>\tfrac{\varphi(7^{z})}{\varphi(7^{z+1})} = \tfrac{7^{z-1}\varphi(7)}{7^{z}\varphi(7)} = \tfrac{1}{7}</math>.</LI></UL> | ||
+ | Thus, the sum of all possible values of <math>\tfrac{\varphi(a)}{\varphi(b)}</math> is<cmath>(4+8)(\tfrac{1}{6}+\tfrac{1}{9})(4+5)(\tfrac{1}{6}+\tfrac{1}{7}) = \frac{65}{7}.</cmath>We extract <math>65+7 = \boxed{72}</math>. |
Revision as of 21:50, 9 September 2025
Problem
Positive integers and
satisfy
. The sum of all possible values of
is
where
and
are relatively prime positive integers. Find
.
Solution
Let and
, where
,
,
, and
are all nonnegative integers and
is a positive integer not divisible by any of
,
,
, and
. Then,
Now, we determine the possible values of each factor on the right hand side based on the values of
,
,
, and
.
- If
, then
; if
, then
.
- If
, then
; if
, then
.
- If
, then
; if
, then
.
- If
, then
; if
, then
.
Thus, the sum of all possible values of is
We extract
.